Sorry if the photo is sideways, can someone please help me

Sorry If The Photo Is Sideways, Can Someone Please Help Me

Answers

Answer 1

The length of AB is approximately 12.704 units.

How to find the length?

To solve this problem, we can use trigonometry and the fact that the easel forms a 30° angle to find the length of AB.

According to given information:

We know that RC is 22, and that angle R is 30°. Let's use the trigonometric function tangent to find AB:

tan(30°) = AB / RC

We can rearrange this equation to solve for AB:

AB = tan(30°) * RC

Using a calculator or trigonometric table, we find that tan(30°) = 0.5774 (rounded to four decimal places). Therefore:

AB = 0.5774 * 22

AB ≈ 12.704

So the length of AB is approximately 12.704 units.

to know more about function visit:

brainly.com/question/12431044

#SPJ1


Related Questions

Let S be the part of the plane 3+ + 2) + z = 1 which lies in the first octant, oriented upward. Use the Stokes theorem to find the flux of the vector field F = 3i+3j + 4k across the surface S.

Answers

The surface integral of the dot product between the vector field F = 3i + 3j + 4k and the unit normal vector of the surface S is equal to zero.

To use Stokes' theorem to find the flux of the vector field F = 3i + 3j + 4k across the surface S, which is the part of the plane 3x + 2y + z = 1 in the first octant and oriented upward.

Stoke's theorem statement is “the surface integral of the curl of a function over the surface bounded by a closed surface will be equal to the line integral of the particular vector function around it.” Stokes theorem gives a relation between line integrals and surface integrals.

First, we need to parametirize the curve C that bounds the surface S. Since S is in the first octant, x, y, and z are all non-negative.

The boundary C consists of three line segments: (i) from (0, 0, 0) to (1/3, 0, 0), (ii) from (1/3, 0, 0) to (0, 1/2, 0), and (iii) from (0, 1/2, 0) to (0, 0, 0). Next, calculate the curl of F, which is the cross product of the del operator and F:
curl(F) = (∂Fz/∂y - ∂Fy/∂z)i - (∂Fx/∂z - ∂Fz/∂x)j + (∂Fy/∂x - ∂Fx/∂y)k = (0 - 0)i - (0 - 0)j + (0 - 0)k = 0.

Since curl(F) = 0, the line integral of F over C is also 0.

According to Stokes' theorem, the flux of F across S equals the line integral of F over C, which we found to be 0.

Therefore, the flux of the vector field F = 3i + 3j + 4k across the surface S is 0.

Learn More About Stoke's Theoram: https://brainly.com/question/17256782

#SPJ11

Which of the following is an even function? g(x) = (x – 1)2 1 g(x) = 2x2 1 g(x) = 4x 2 g(x) = 2x

Answers

The only even function among the given options is g(x) = 2x^2, so the answer is B) g(x) = 2x^2.

A function is even if it satisfies the property g(-x) = g(x) for all x.

Checking each of the given functions:

g(x) = (x - 1)^2 is not even, because g(-x) = (-x - 1)^2 = x^2 + 2x + 1, which is not equal to g(x) = (x - 1)^2.

g(x) = 2x^2 is even, because g(-x) = 2(-x)^2 = 2x^2 = g(x) for all x.

g(x) = 4x^2 is even, because g(-x) = 4(-x)^2 = 4x^2 = g(x) for all x.

g(x) = 2x is odd, because g(-x) = 2(-x) = -2x = -g(x) for all x.

Therefore, the only even function among the given options is g(x) = 2x^2, so the answer is B) g(x) = 2x^2.

To know more about even function refer here:

https://brainly.com/question/23446734

#SPJ11

Let f:R → R be a function that satisfies ∫f(t)dt then the value of f(log e 5) is

Answers

Unfortunately, I cannot provide an answer to this question as it is incomplete. The given information ∫f(t)dt is not enough to determine the value of f(log e 5). More information about the function f would be needed, such as its explicit form or additional properties. Please provide more context or information to help me answer your question accurately.
Given that f is a function f:R → R that satisfies ∫f(t)dt, we need to find the value of f(log e 5).

By definition, log e 5 is the natural logarithm of 5, which can be written as ln(5). Therefore, we want to find the value of f(ln(5)).

However, without further information on the function f or the integral bounds, it's not possible to determine the exact value of f(ln(5)). Please provide more details about the function or the integral to get a specific answer.

Learn more about integration here: brainly.com/question/18125359

#SPJ11

How to find the area of this whole figure? Please help me

Answers

The area of the whole figure is 31.5 sq. units.

What is the area of a figure?

The area of a given figure connotes its expanse in a 2 dimensional plane. The shape and size of a given figure determines how to calculate its area.

From the given question, the figure given can be likened to a rhombus. So that;

area of a rhombus = (diagonal 1 * diagonal 2)/ 2

Then,

area of the figure = (diagonal 1 * diagonal 2)/ 2

where: diagonal 1 = 7.5, and diagonal 2 = 8.4

So that;

area of the figure = (7.5*8.4)/ 2

                             = 63/ 2

                             = 31.5

The area of the whole figure is 31.5 sq. units.

Learn more about area of a figure at https://brainly.com/question/30944216

#SPJ1

A magazine listed the number of calories and sodium content​ (in milligrams) for 13 brands of hot dogs. Examine the​ association, assuming that the data satisfy the conditions for inference. Complete parts a and b

Answers

Option B is correct. In this context, the meaning is: Among hot dogs with the same number of calories, the sodium content varies, with a standard deviation of about 75 milligrams

The calculated test statistic is 3.75

How to get the correct option

The test statistics can be gotten from the data that we already have available in this question

The coefficient is given as 2.235

The Standard error of the coefficient is given as 0.596

The formula used is given as

Such that t = coefficient /  Standard error

where the coefficient = 2.235

The standard error = 0.596

We have to apply the values to formula:

= 2.235 / 0.596

= 3.75

Read more on t statistic here:

https://brainly.com/question/6589776

#SPJ1

I NEED HELP PLEASE!

1. 3 statements about limiting frictional force between two surfaces are given below.


A - Nature of surfaces in contact affects to limiting frictional force.

B - Normal reaction between them affects to limiting frictional force.

C - Area of surfaces in contact affects to limiting frictional force.


Correct statement / statements from above A, B, C is/ are,

(1) A

(2) B

(3) A and C

(4) A, B and C

Answers

The correct statement is (3) A and C.

The limiting frictional force between two surfaces is dependent on the nature of the surfaces in contact and the area of the surfaces in contact. The normal reaction between them is not a factor affecting the limiting frictional force.
Let's analyse the options:Option 1: Statement A is correct. The nature of surfaces in contact determines the friction between them. Rough and irregular surfaces have higher friction than smooth surfaces.Option 2: Statement B is incorrect. The normal reaction between the surfaces does not affect the limiting frictional force. Only the maximum static friction depends on the normal reaction.Option 3: Statements A and C are correct. Both nature of surfaces and area of contact determine the friction between them.Option 4: All statements A, B and C are incorrect. Nature of surfaces, normal reaction and area of contact affect the limiting frictional force between two surfaces.

The limiting frictional force depends only on:

A. The nature of surfaces in contact: Rough and irregular surfaces have higher friction than smooth surfaces. C. The area of surfaces in contact: Larger the contact area, higher is the friction between the surfaces.(3) A and C is the right option

When solving two-step equations, you are using the reverse order of operations to solve the two-step equations.

Select one:
True
False

Answers

Therefore , the solution of the given problem of equation comes out to be False.

What is an equation?

In order to demonstrate consistency between two opposing statements, variable words are frequently used in sophisticated algorithms. Equations are academic phrases that are used to demonstrate the equality of different academic figures. Consider expression the details as y + 7 offers. In this case, elevating produces b + 7 when partnered with building y + 7.

Here,

False.

The order of operations we use to solve two-step equations is the same order we use to solve every other mathematical statement,

which is commonly recalled by the acronym PEMDAS. (parentheses, exponents, multiplication and division from left to right, and addition and subtraction from left to right).

The fundamental distinction is that we are carrying out the operations against the equation's representation. For instance, consider the following equation:

=> 2x + 5 = 11

To get the following result, we would first subtract 5 from both sides, then divide by 2.

=> x = 3

In order to "undo" the operations that were carried out on the variable in the original equation, we are utilising the same series of operations as usual, but in reverse order.

To know more about equation visit:

https://brainly.com/question/649785

#SPJ1

For y = 72√x, find dy, given x = 4 and Δx = dx = 0.21
dy = (Simplify your answer.)

Answers

To find dy for the function y = 72√x, given x = 4 and Δx = dx = 0.21, we will first find the derivative of y with respect to x and then plug in the given values.

1. Differentiate y with respect to x: y = 72√x can be rewritten as y = 72x^(1/2)
  Apply the power rule: dy/dx = 72 * (1/2)x^(-1/2)
  Simplify: dy/dx = 36x^(-1/2)

2. Plug in the given values: x = 4 and dx = 0.21
  dy/dx = 36(4)^(-1/2)
  dy/dx = 36(1/√4)
  dy/dx = 36(1/2)
  dy/dx = 18

3. Calculate dy: dy = (dy/dx) * dx
  dy = 18 * 0.21
  dy = 3.78

So, for y = 72√x, dy is 3.78 when x = 4 and Δx = dx = 0.21.

To learn more about ''find dy, given x'' visit : https://brainly.com/question/15272082

#SPJ11

Can someone help me please

Answers

Answer:

A

Step-by-step explanation:

Simple interest gains interest only on the principal sum and so each year has the same interest. So, this example is not simple interest. Compound interest is calculated on the principal and the accumulated interest of the previous years. So, this is compound interest.

      Interest is 4%.

      [tex]A = P*(1 + R \%)^n[/tex]

Here A is the amount we receive after n years, P is the principal and R is the rate of interest.

        A = $1300 , R = 4%

            [tex]1300 = P * (1+0.04})\\\\1300 = P * 1.04\\\\\dfrac{1300}{1.04}=P\\\\\\\dfrac{130000}{104}=P[/tex]

            P = $ 1250

PLEASE HELP WILL GIVE BRAINLEST !!!!

Answers

The answer is the first one.

The probability that Sam parks in a no-parking zone and gets a parking ticket is 0. 07,and the probability that Sam Connor finr a legal parking space and has to park in the no-parking zone is 0. 50. On Monday,Sam arrives at school and has to park in a no-parking zone. Find the probability that he will get a parking ticket.

Answers

The probability that Sam will get a parking ticket given that he has to park in a no-parking zone on Monday is approximately 0.1308 or 13.08%.

We can use Bayes' theorem to solve this problem. Let A be the event that Sam gets a parking ticket and B be the event that Sam parks in a no-parking zone. Then, we want to find P(A|B), which is the conditional probability of A given B.

Bayes' theorem states that P(A|B) = P(B|A)*P(A)/P(B), where P(B|A) is the probability of B given A, P(A) is the prior probability of A, and P(B) is the prior probability of B.

From the problem, we know that P(A) = 0.07, P(B|A) = 1 (since if Sam parks in a no-parking zone, he will definitely get a parking ticket), and P(B|not A) = 0.50 (since if Sam finds a legal parking space, he has a 0.50 probability of parking in a no-parking zone).

To find P(B), we can use the law of total probability, which states that P(B) = P(B|A)*P(A) + P(B|not A)*P(not A), where P(not A) = 1 - P(A).

Therefore, P(B) = 10.07 + 0.50(1-0.07) = 0.5351.

Finally, we can use Bayes' theorem to find P(A|B):

P(A|B) = P(B|A)P(A)/P(B) = 10.07/0.5351 ≈ 0.1308.

Therefore, the probability that Sam will get a parking ticket given that he has to park in a no-parking zone on Monday is approximately 0.1308 or 13.08%.

To know more about probability, refer here:

https://brainly.com/question/30034780s#

#SPJ11

Here is a set of data showing the test scores for US History class:


56, 88, 70, 72, 90, 85, 99, 65, 66, 54, 74, 85, 91, 92, 72, 88, 97, 62, 88 Create a stem and leaf plot to show this data. Hint: Decide how many stems you will need.


can somebody help me ?

Answers

Hi! I'd be happy to help you create a stem and leaf plot using the provided set of data for US History class test scores.

Step 1: Arrange the data in ascending order.


54, 56, 62, 65, 66, 70, 72, 72, 74, 85, 85, 88, 88, 88, 90, 91, 92, 97, 99

Step 2: Determine the range of the data.


The data ranges from 50s to 90s, so we will need 5 stems: 5, 6, 7, 8, and 9.

Step 3: Create the stem and leaf plot using the stems and corresponding leaves (the units digits of the data).

5 | 4 6
6 | 2 5 6
7 | 0 2 2 4
8 | 5 5 8 8 8
9 | 0 1 2 7 9

Here is the completed stem and leaf plot for the US History class test scores. The stems represent the tens digits (50s, 60s, 70s, 80s, 90s), and the leaves represent the units digits of the scores in each range.

To Know more about  create refer here

https://brainly.com/question/29853117#

#SPJ11

Every time you practice, you gain more skills.



Conditional:


Hypothesis:


Conclusion:


Converse:


Inverse:


Contrapositive:

Answers

Hypothesis: Every time you practice, you gain more skills.

What happens when you practice?

Conclusion: Gain of skills is a result of practice.

Converse: If you gain more skills, then you practice every time.

Inverse: If you don't practice, then you don't gain more skills.

Contrapositive: If you don't gain more skills, then you don't practice every time.

The hypothesis states that practicing leads to an increase in skills. This can be interpreted as a cause and effect relationship between the two variables.

The conclusion reiterates that gaining skills is a result of practice.

The converse of the statement flips the order of the hypothesis and the conclusion. It states that if you gain more skills, then you must have practiced every time.

This may not be entirely true because there can be other factors that contribute to the gain of skills besides practice.

The inverse of the statement negates both the hypothesis and the conclusion. It states that if you don't practice, then you don't gain more skills.

This statement is true because practice is a necessary condition for gaining skills. However, it doesn't mean that practicing alone guarantees the gain of skills.

The contrapositive of the statement flips the order of the negated hypothesis and the negated conclusion. It states that if you don't gain more skills, then you didn't practice every time.

This statement is also true because if one doesn't practice, they cannot expect to gain more skills.

Learn more about practice

brainly.com/question/2962699

#SPJ11

Use a calculator to find the values of the inverse trigonometric functions. Round to the nearest degree.

Answers

The values of the inverse trigonometric functions are 72°, 76° and 85°.

How to explain the steps

The range of the inverse trigonometric function is limited to a certain interval based on the domain of the original trigonometric function.

Its also important to identify the trigonometric ratio that corresponds to the given value.

The value on degree for inverse of sin (2/3) will be 41.81° which is 42°. Also, inverse of tan(4) is 76° using the calculator.

Learn more about trigonometry on

https://brainly.com/question/24349828

#SPJ1

Use a calculator to find the values of the inverse trigonometric functions. Round to the nearest degree.

inverse of sin (2/3)

inverse of tan(4)

inverse of tan (0.1)

• Orhan studied the relationship between


temperature and sales of refreshments


at the concession stands inside the football


stadium. He wrote an equation for the


linear function that relates temperature (x)


and refreshment sales (y). Which of the


following could be Orhan's equation?


A. Y=3x2 + 25


B. Y = 15x + 40


C. Y= llx - 55


-


D. Y= x – 135

Answers

The equation that could be Orhan's equation for the linear function that relates temperature and refreshment sales is   Y = 15x + 40.

This is because the equation is in the form of y = mx + b, where m is the slope (or rate of change) and b is the y-intercept. In this case, the slope is 15, which means that for every increase of 1 degree in temperature, there will be an increase of 15 units in refreshment sales.

The y-intercept is 40, which means that even at a temperature of 0 degrees, there will still be some refreshment sales (40 units).

The other equations do not have a linear relationship between temperature and sales, as they either have a quadratic term (A), a negative slope (C), or a large negative constant term (D).

Hence, option B is the correct answer.

To know more about Orhan's equation refer here:

https://brainly.com/question/27465710

#SPJ11

A health insurance company wants to know the proportion of admitted hospital patients who have a type 2 diabetes at orlando health hospitals. how large of a sample should be taken to estimate the proportion within 6% at 93% confidence?

Answers

To estimate the proportion of admitted hospital patients who have type 2 diabetes at Orlando Health Hospitals within a certain margin of error and confidence level,

we can use the following formula to determine the necessary sample size:

[tex]n = [(Z^2 * p * q) / E^2] / [1 + ((Z^2 * p * q) / E^2N)][/tex]

where:

n = sample size

Z = z-score for the desired confidence level (in this case, 1.81 for a 93% confidence level)

p = estimated proportion of patients with type 2 diabetes (unknown)

q = 1 - p

E = margin of error (0.06)

N = population size (unknown)

Since we do not know the estimated proportion of patients with type 2 diabetes or the population size,

we can assume a conservative estimate of p = q = 0.5, which maximizes the sample size required.

Plugging in the values, we get:

[tex]n = [(1.81^2 * 0.5 * 0.5) / 0.06^2] / [1 + ((1.81^2 * 0.5 * 0.5) / 0.06^2N)][/tex]

Simplifying, we get:

[tex]n = 1242.95 / [1 + (2.4 / N)][/tex]

To satisfy the requirements of the problem, we need to round up to the nearest whole number, so we need a sample size of at least 1243 patients.

Note that if we had a better estimate for p or N, we could use those values in the formula to get a more precise sample size.

To know more about margin of error refer here

https://brainly.com/question/29101642#

#SPJ11

The point (7,8) in the coordinate plane represents a ratio. adela claims that you can find equivalent ratio by adding the same number to both coordinate of the point. is adela correct? explain.

Answers

For the point (7,8) in the coordinate plane which represents a ratio, Adela claims that you can find equivalent ratio by adding the same number to both coordinate of the point is incorrect.

Adela claim is not correct. To find an equivalent ratio, you should multiply (or divide) both coordinates by the same nonzero number instead of adding the same number.

1. The point (7,8) represents the ratio 7:8.

2. If we add the same number to both coordinates, let's say 2, we get the point (9,10), which represents the ratio 9:10.

3. We can check if 7:8 and 9:10 are equivalent ratios by cross-multiplying:

7 * 10 = 70 and 8 * 9 = 72. Since 70 ≠ 72, these ratios are not equivalent.

Therefore, Adela's claim is incorrect because adding the same number to both coordinates of the point does not result in an equivalent ratio. To find equivalent ratios, you should multiply (or divide) both coordinates by the same nonzero number.

Learn more about Equivalent ratio:

https://brainly.com/question/2914376

#SPJ11

The cone is formed from 3,200 ft3 of gravel. If the height of the cone is 24 feet, what is the radius, in feet, of the base of the cone? Use the π button on your calculator to determine the answer. Round your answer to the nearest tenth of afoot. The radius of the base of the cone is approximately ____ feet

Answers

The radius of the base of the cone is approximately 12.65 feet if The cone is formed from 3,200 ft of gravel.

Height of cone =  24 feet

The volume of the cone = [tex]3,200 ft^3[/tex]

To find the volume of the cone, the formula used here is:

V =π* [tex]r^2h[/tex]

Here, the values of V and H are known terms. we need to calculate the radius r of the cone.

π = 3.14 constant value

Substituting the values in the above equation, we get:

[tex]3,200 = (1/3)^2*(24)*[/tex] π

[tex]r^2[/tex]= 3,200 / (8π)

[tex]r^2[/tex] = 400 / π

r = 12.65

Therefore, we can conclude that the radius of the base of the cone is  12.65 feet.

To learn more about Volume of Cone

https://brainly.com/question/29767724

#SPJ4

Is the triangle similar to PQR? State whether each triangle similar to PQR by answering yes or no

Answers

Yes, all three triangles RQS, QSR, and PRS are similar to triangle PQR.

Describe Congruency of triangle?

Congruency of triangles refers to the condition in which two or more triangles have the same size and shape. In other words, if two or more triangles have congruent sides and angles, then they are said to be congruent.

The criteria for determining the congruency of triangles are based on the properties of sides and angles. There are various ways to show that two triangles are congruent, including the following:

Side-Side-Side (SSS) Congruence: If the three sides of one triangle are equal to the three sides of another triangle, then the two triangles are congruent.

Side-Angle-Side (SAS) Congruence: If two sides and the angle between them in one triangle are equal to two sides and the angle between them in another triangle, then the two triangles are congruent.

Angle-Side-Angle (ASA) Congruence: If two angles and the side between them in one triangle are equal to two angles and the side between them in another triangle, then the two triangles are congruent.

Angle-Angle-Side (AAS) Congruence: If two angles and a side not between them in one triangle are equal to two angles and the corresponding side not between them in another triangle, then the two triangles are congruent.

Hypotenuse-Leg (HL) Congruence: If the hypotenuse and one leg of a right triangle are equal to the hypotenuse and one leg of another right triangle, then the two triangles are congruent.

Yes, all three triangles RQS, QSR, and PRS are similar to triangle PQR.

To know more about triangle visit:

https://brainly.com/question/21832406

#SPJ1

Solve problems 1 and 4 ONLY with the rules given on the paper.

Answers

The solution to the equations obtained using inverse trigonometric function values are;

1. x ≈ 0.65

4. x ≈ 0.95

What are trigonometric functions?

Trigonometric functions indicates the relationships between the angles in a right triangle and two of the sides of the triangle. Trigonometric functions are periodic functions.

The value of x is obtained from the inverse trigonometric function of the output value of the trigonometric function, as follows;

The inverse function for sine is arcsine

The inverse function for cosine is arccosine

The inverse function for the tangent of an angle is arctangent

1. sin(x) = 0.6051

Therefore; x = arcsine(0.6051) ≈ 0.65 radians

The value of x in the interval [0·π, 2·π] is x ≈ 0.65

4. tan(x) = 1.3972

Therefore, x = arctan(1.3972) ≈ 0.95

Learn more on inverse trigonometric functions here: https://brainly.com/question/13276558

#SPJ1

In the formula


A(t) = Pert for continuously compound interest, the letters P, r, and t stand for ---Select--- percent interest prime rate amount after t years principal number of years , ---Select--- interest rate per year rate of return investment amount investment per year interest rate per day , and ---Select--- number of months number of days number of time periods number of years number of times interest is compounded per year , respectively, and A(t) stands for ---Select--- amount of principal amount after t days amount of interest earned after t years amount of interest earned in year t amount after t years. So if $200 is invested at an interest rate of 4% compounded continuously, then the amount after 3 years is $. (Round your answer to the nearest cent. )

Answers

In the formula [tex]A(t) = Pe^{rt}[/tex] continuously compound interest P, r, and t stands for Principal, rate of interest, and time respectively, and A(t) stands for Amount after t amount of time.  If $200 is invested at an interest rate of 4% compounded continuously, then the amount after 3 years is $225.5.

The formula for Compound Interest at a continuous period of time is denoted by [tex]A(t) = Pe^{rt}[/tex]

where the Principal amount is multiplied by the exponential value of the interest rate and time passed.

Hence we are given here

P = $200, r = 4% = 0.04, and the amount to be calculated for t = 3 years

Hence we will find the amount by replacing these values to get

A(3) = 200 × e⁰°⁰⁴ ˣ ³

= $200 × e⁰°¹²

= $225.499

rounding it off to the nearest cent gives us

$225.5

To learn more about Compound Interest visit

https://brainly.com/question/30761870

#SPJ4

Correct Question

In the formula [tex]A(t) = Pe^{rt}[/tex] continuously compound interest P, r, and t stands for ______ , _______ , and __________ respectively, and A(t) stands for _______ .

So if $200 is invested at an interest rate of 4% compounded continuously, then the amount after 3 years is $__________. (Round your answer to the nearest cent.)

PLease help 1 and 2 Pythagorean Theorem and if you can explain please

Answers

Answer:

Step-by-step explanation:

The Pythagorean theorem has the formula a squared(leg) + b squared(leg) = c squared(longest leg). This means [tex]12^{2} +16^{2} =20^{2} -- > 144+256=400[/tex] which is true meaning number 1 is a right triangle. [tex]10^{2} +49.5^{2} = 50.5^{2} -- > 100+2450.25=2550.25[/tex] is true meaning number 2 is also a right triangle because the sum of the shortest legs squared are equal to the longest leg (hypotenuse) squared.

Solve the equation and check your solution: x + 4 = -2 + x

Answers

The equation x + 4 = -2 + x has no solution for x

Solving the equation and checking the solution

From the question, we have the following parameters that can be used in our computation:

x + 4 = -2 + x

Subtract x from both sides of the equation

so, we have the following representation

x - x + 4 = -2 + x - x

When the like terms of the equation are evaluated, we have

4 = -2

The above equation is false

This is because 4 and -2 do not have the same value

Hence, the equation has no solution

Read more about equation at

https://brainly.com/question/18831322

#SPJ1

The function f(x) = 2x + 7x{-1} has one local minimum and one local maximum. This function has a local maximum at x = with value and a local minimum at x = with value

Answers

The function has a local maximum at x = -√(2/7) with value -3√14 and a local minimum at x = √(2/7) with value 3√14.

To find the local maximum and minimum of the function f(x) = 2x + 7x⁻¹, we need to find the critical points of the function and then use the second derivative test to determine if they are local maxima or minima.

First, we find the derivative of f(x):

f'(x) = 2 - 7x⁻²

Setting f'(x) = 0, we get:

2 - 7x⁻² = 0

Solving for x, we get:

x = ±√(2/7)

Next, we compute the second derivative of f(x):

f''(x) = 14x⁻³

At x = ±√(2/7), we have:

f''(±√(2/7)) = ±∞

Since f''(±√(2/7)) has opposite signs at the critical points, ±√(2/7), we conclude that f(x) has a local maximum at x = -√(2/7) and a local minimum at x = √(2/7).

To find the values of the local maximum and minimum, we plug them into the original function:

f(-√(2/7)) = 2(-√(2/7)) + 7/(-√(2/7)) = -3√14

f(√(2/7)) = 2(√(2/7)) + 7/(√(2/7)) = 3√14

Therefore, the function has a local maximum at x = -√(2/7) with value -3√14 and a local minimum at x = √(2/7) with value 3√14.

To learn more about function visit: https://brainly.com/question/12431044

#SPJ11

A cylindrical jar of peanut butter has a height of 4 inches and a diameter of 3 inches. How many cubic inches of peanut butter can the jar hold? Use π = 3.14.

28.26 in3
37.68 in3
113.04 in3
150.72 in3

Answers

The jar can hold 28.26 cubic inches of peanut butter

How to calculate the amount of cubic inches of peanut butter?

The first step is to write out the parameters

A cylindrical jar of peanut has a height of 4 inches

The diameter is 3 inches

The next step is to calculate the radius, this is done by dividing the diameter by 2

radius= 3/2

= 1.5

The formula used to calculate the cubic inches of peanut butter is

V= πr²h

= 3.14 × 1.5² × 4

= 3.14 × 2.25 × 4

= 28.26

Hence the jar can hold 28.26 cubic inches of peanut butter

Read more on cubic inches here
https://brainly.com/question/30963082


#SPJ1

On Friday, Jacob planted a pinto bean in science class. When he returned to school on Monday, the bean had sprouted a stem that was 3 millimeters long. At the end of the week, Jacob's bean sprout had a stem that was 42 millimeters long. How many centimeters did Jacob's bean sprout grow during the week?

Answers

Jacob's bean sprout grew 3.9 centimeters during the week.

What is measurements?

Measurements in math involve the assignment of numerical values to physical quantities, such as length, area, volume, mass, time, temperature, and so on. Measuring objects or events allows us to compare and quantify them, and is an essential part of mathematical problem-solving, as well as many other fields of study

Jacob's bean sprout grew 42 millimeters - 3 millimeters = 39 millimeters during the week.

To convert millimeters to centimeters, we need to divide by 10 since there are 10 millimeters in 1 centimeter.

So, the growth in centimeters is 39 millimeters ÷ 10 = 3.9 centimeters.

Therefore, Jacob's bean sprout grew 3.9 centimeters during the week.

To learn more about measurements from the given link:

https://brainly.com/question/4725561

#SPJ4

AH = Actual Hours SH = Standard Hours AR = Actual Rate SR = Standard Rate Compute the direct labor rate and efficiency variances for the period and classify each as favorable, unfavorable or no variance

Answers

To compute the direct labor rate and efficiency variances, we will use the given terms: Actual Hours (AH), Standard Hours (SH), Actual Rate (AR), and Standard Rate (SR). Here's a step-by-step explanation:

Step 1: Calculate the Actual Labor Cost
Actual Labor Cost = AH * AR

Step 2: Calculate the Standard Labor Cost
Standard Labor Cost = SH * SR

Step 3: Calculate the Labor Rate Variance
Labor Rate Variance = (AR - SR) * AH

Step 4: Classify the Labor Rate Variance
If the Labor Rate Variance is positive, it is unfavorable. If it is negative, it is favorable. If it is zero, there is no variance.

Step 5: Calculate the Standard Labor Cost for Actual Hours
Standard Labor Cost for Actual Hours = AH * SR

Step 6: Calculate the Labor Efficiency Variance
Labor Efficiency Variance = (AH - SH) * SR

Step 7: Classify the Labor Efficiency Variance
If the Labor Efficiency Variance is positive, it is unfavorable. If it is negative, it is favorable. If it is zero, there is no variance.

By following these steps, you can compute the direct labor rate and efficiency variances for the period and classify each as favorable, unfavorable, or no variance.

To know more about efficiency variances refer here

https://brainly.com/question/28240324#

#SPJ11

James weighs 8712 pounds. he has 2 dogs that each weigh 1314 pounds. how many more pounds does james weigh than both of his dogs combined?

Answers

James weighs 6084 more pounds than both of his dogs combined.

To find out how many more pounds James weighs than both of his dogs combined, we first need to calculate the total weight of the dogs. Since he has two dogs that weigh 1314 pounds each, we can find the total weight of the dogs by multiplying 1314 by 2, which gives us 2628 pounds.

Next, we can add the weight of both dogs together to get the total weight of the dogs, which is 2628 pounds. We can then subtract the weight of the dogs (2628 pounds) from James' weight (8712 pounds) to find out how many more pounds James weighs than both of his dogs combined.

Therefore, James weighs 6084 more pounds than both of his dogs combined. This can be calculated by subtracting the weight of the dogs (2628 pounds) from James' weight (8712 pounds), which gives us 6084 pounds.

To know more about weight, refer to the link below:

https://brainly.com/question/8308591#

#SPJ11

143°
(8x+55)° find the value of X

Answers

Answer:11

Step-by-step explanation:Vertically opposite angles are the same.
Alternate(Z)angles are the same.Therefore we have an angle of 143 vertically opposite to the equation.143-55=88
88/8=11

Suppose a bus arrives at a bus stop every 26 minutes. If you arrive at the bus stop at a random time, what is the probability that you will have to wait at least 4 minutes for the bus?

Answers

The time between buses arriving at the stop follows an exponential distribution with a mean of 26 minutes.

To find the probability of waiting at least 4 minutes for the bus, we can use the cumulative distribution function (CDF) of the exponential distribution:

P(waiting at least 4 minutes) = 1 - P(waiting less than 4 minutes)

The probability of waiting less than 4 minutes can be calculated using the CDF:

P(waiting less than 4 minutes) = 1 - e^(-4/26) ≈ 0.146

Therefore, the probability of waiting at least 4 minutes for the bus is:

P(waiting at least 4 minutes) = 1 - 0.146 ≈ 0.854

So the probability of having to wait at least 4 minutes for the bus is about 85.4%.

Other Questions
007 10.0 points An advertisement is run to stimulate the sale of cars. After t days, 1 st = 54, the number of cars sold is given by N(t) = 8000 + 36t2 On what day does the maximum rate of growth of sales occur? 1. on day 12 2. on day 11 3. on day 14 4. on day 10 5. on day 13 Which aspects of the compromise 1850 were a victory for the south and why? What volume of 16. 2 M NH3 is required to prepare 350. 0 mL of 0. 200 M NH3 Express this number in scientific notation.9 ten thousandths Each deck of cards in a a box has a weight of 3.4 oz.the box contains 64 decks of cards.what is the total weight of the cards inside the box?teh oz are rounded to the nearest oz What physiological adaptation helps annual plants whose climate has long periods of drought followed by brief periods of rain? Name two things for which the president needs the consent of the Senate. On the average the time spent by college students every week on computer gaming is 15 hours with a standard deviation 3. a random sample of 350 students were taken. find the best point estimated of the population mean and 95% confidence interval for the population mean HELP PLS!! I AM LACKING BRAIN CELLS RN!! :( Complete the balanced molecular reaction for the following weak acid with a strong base: HNO2(aq) + Ca(OH)2 (aq) ->Correct answer should be 2 HNO2(aq) + Ca(OH)2(aq) -> 2 H2O(l) + Ca(NO2)2(aq). Why? Find a pair of integers whose difference gives zero. A)8 and 1 B)8 and 2 C)8 and 3 D)-8 and -8 A box of tissues is shaped like a rectangular prism and has a volume of 288 cubic inches. A uniform, 6 m long and 600-N beam, rests on two supports, as shown. The force exerted onthe b eam by the right support B is closest to: Which definition describes a scientific theory?OA. A claim, assumption, or method that is commonly thought to bebased on science but is not scientificB. The pursuit of knowledge about the natural world and theapplication of that knowledgeC. An explanation for a natural phenomenon that is based on factsand is supported by the results of many experimentsD. A method of carefully examining an idea, using a variety ofthought processes to construct and evaluate knowledge Calculate how many formula units of sodium hydroxide are present in 16. 0g of NaOH. From your answer, deduce how many sodium ions (Na') and hydroxide ions (OH) are present in this mass of sodium hydroxideuhhhh guys pls help Your received information that your brother was involved in a fight.write a letter to him to be patient and tolerant What baked good is a good friday tradition in england. Pleasee helpf(x) = 3x - 7x + 4f(2)= [?] Four point masses 2kg, 4kg, 6kg and 8kg are placed at the corners of Square ABCD of 2cm long respectively. Find the Position of centre of mass of the system from the corner A. help pls. my mom is mad bc i dont have this done.